Think about 5

Algebra Level 3

( x + 3 ) ( x + 4 ) ( x + 6 ) ( x + 7 ) = 1120 \large (x+3)(x+4)(x+6)(x+7) = 1120

Find the sum of all reals x x satisfying the equation above.


The answer is -10.

This section requires Javascript.
You are seeing this because something didn't load right. We suggest you, (a) try refreshing the page, (b) enabling javascript if it is disabled on your browser and, finally, (c) loading the non-javascript version of this page . We're sorry about the hassle.

23 solutions

Chew-Seong Cheong
Mar 30, 2015

Simpler solution by @XerDazzle XD酱 as follows:

( x + 3 ) ( x + 4 ) ( x + 6 ) ( x + 7 ) = 1120 = 4 × 5 × 7 × 8 (x+3)(x+4)(x+6)(x+7) = 1120 = 4 \times 5 \times 7 \times 8 { x = 1 ( 1 + 3 ) ( 1 + 4 ) ( 1 + 6 ) ( 1 + 7 ) = 4 × 5 × 7 × 8 x = 11 ( 11 + 3 ) ( 11 + 4 ) ( 11 + 6 ) ( 11 + 7 ) = ( 8 ) ( 7 ) ( 5 ) ( 4 ) \implies \begin{cases} x = 1 & \implies (1+3)(1+4)(1+6)(1+7) = 4 \times 5 \times 7 \times 8 \\ x = -11 & \implies (-11+3)(-11+4)(-11+6)(-11+7) = (-8)(-7)(-5)(-4) \end{cases}

Therefore, sum of solutions is 1 11 = 10 1-11 = \boxed{-10} .


Previous solution

Let y = x + 5 y = x+5 (see note), then:

( x + 3 ) ( x + 4 ) ( x + 6 ) ( x + 7 ) = 1120 ( y 2 ) ( y 1 ) ( y + 1 ) ( y + 2 ) = 1120 ( y 2 1 ) ( y 2 4 ) = 1120 y 4 5 y 2 + 4 = 1120 y 4 5 y 2 1116 = 0 ( y 2 + 31 ) ( y 2 36 ) = 0 y = ± 6 x = { 6 5 = 11 6 5 = 1 \begin{aligned} (x+3)(x+4)(x+6)(x+7) & = 1120 \\ \color{#D61F06}{(y-2)} \color{#3D99F6}{(y-1)}\color{#3D99F6}{(y+1)}\color{#D61F06}{(y+2)} & = 1120 \\ \color{#3D99F6}{(y^2-1)}\color{#D61F06}{(y^2-4)} & =1120 \\ y^4 - 5y^2 + 4 &= 1120 \\ y^4 - 5y^2 -1116 &= 0 \\ (y^2+31)(y^2-36) & = 0 \\ \Rightarrow y & = \pm 6 \\ \Rightarrow x & = \begin{cases} -6-5 & = -11 \\ 6-5 & = 1 \end{cases} \end{aligned}

Therefore the sum of x x satisfying the equation = 11 + 1 = 10 = -11+1 = \boxed{-10}


Note that 5 is the arithmetic mean of 3, 4, 6 and 7. Using y = x + 5 y=x+5 results in the four factors y ± 1 y \pm 1 and y ± 2 y \pm 2 , so that we eventually get a quadratic equation y 4 5 y 2 + 4 = 1120 y^4 - 5y^2 + 4 = 1120 without the y 3 y^3 and y y terms, which can be easily solved.

This one's a lot simpler than the first one....

Shreyas Pai - 5 years, 6 months ago

Log in to reply

First one is Just more work, but the 2nd one needs the insight for choosing y = x + 5

Peter van der Linden - 4 years, 10 months ago

Awesome solution

genis dude - 4 years, 9 months ago

Log in to reply

Interested solution 👌

Oghenemine Ofejiro N. - 9 months ago

why do you take y=x+5 , why not x+4 or something else , can you brief me a little bit ?

Syed Hissaan - 4 years, 9 months ago

Log in to reply

So that I get ( y 1 ) ( y + 1 ) = y 2 1 (y-1)(y+1) = y^2 - 1 and ( y 2 ) ( y + 2 ) = y 2 4 (y-2)(y+2) = y^2 - 4 . Other options are not possible. Why don't you try y = x + 4 y=x+4 .

Chew-Seong Cheong - 4 years, 9 months ago

Log in to reply

oh , i got that one , thanks .

Syed Hissaan - 4 years, 9 months ago

Because y = x + 5 y=x+5 makes ( x + 3 ) ( x + 7 ) = ( y 2 ) ( y + 2 ) = y 2 4 (x+3)(x+7) = (y-2)(y+2) = y^2-4 and ( x + 4 ) ( x + 6 ) = ( y 1 ) ( y + 1 ) = y 2 1 (x+4)(x+6) = (y-1)(y+1) = y^2-1 , then ( y 2 1 ) ( y 2 4 ) = y 4 5 y 2 + 4 (y^2-1)(y^2-4)= y^4-5y^2+4 so eliminated the x 3 x^3 term and it becomes a quadratic equation which is easily solved.

Chew-Seong Cheong - 3 years, 7 months ago

Log in to reply

Wonderful solution .

Syed Hissaan - 3 years, 7 months ago

But why (x+5)?

Jann Ryan Novela - 4 years, 3 months ago

Log in to reply

Because 5 is not a subtrahend in any of the factors

FLAME85 Li - 3 years, 7 months ago

Because y = x + 5 y=x+5 makes ( x + 3 ) ( x + 7 ) = ( y 2 ) ( y + 2 ) = y 2 4 (x+3)(x+7) = (y-2)(y+2) = y^2-4 and ( x + 4 ) ( x + 6 ) = ( y 1 ) ( y + 1 ) = y 2 1 (x+4)(x+6) = (y-1)(y+1) = y^2-1 , then ( y 2 1 ) ( y 2 4 ) = y 4 5 y 2 + 4 (y^2-1)(y^2-4)= y^4-5y^2+4 so eliminated the x 3 x^3 term and it becomes a quadratic equation which is easily solved.

Chew-Seong Cheong - 3 years, 7 months ago

best solution of all

Saksham Jain - 3 years, 6 months ago

Log in to reply

Glad that you like it.

Chew-Seong Cheong - 3 years, 6 months ago

Elegant solution

Ayachi Younes - 2 years, 6 months ago

IT IS EASY by Factorization! 1120=4×5×7×8 or 1120=-4×-5×-7×-8 so x=1 or -11!( •̀∀•́ )

XerDazzle XD酱 - 2 years, 5 months ago

Log in to reply

You are right.

Chew-Seong Cheong - 2 years, 5 months ago

Log in to reply

Can you explain the step after factorisation in a bit more depth, please?

Sol Rey - 1 year, 10 months ago

Log in to reply

@Sol Rey Note that ( a b ) ( a + b ) = a 2 b 2 (a-b)(a+b) = a^2 - b^2 . Therefore, ( y 1 ) ( y + 1 ) = y 2 1 \color{#3D99F6} (y-1)(y+1) = y^2 - 1 and ( y 2 ) ( y + 2 ) = y 2 4 \color{#D61F06} (y-2)(y+2) = y^2 - 4 ,

Chew-Seong Cheong - 1 year, 10 months ago

Why we ignore “ y^2 + 31=0 ?

Alby Cosmy 99 - 2 years, 5 months ago

Log in to reply

y 2 + 31 = 0 y = ± 31 i y^2 + 31 = 0 \implies y = \pm \sqrt{31} i , not real.

Chew-Seong Cheong - 1 year, 10 months ago

Even simpler: the term on the left is symmetric around x=-5. It crosses the zero 4 times around x=5 and gets more positive for x<-7 and x>-3. It will cross y=1120 at some x1=-5-z and x2=-5+z. Then, x1+x2= -5-z+(-5+z)=-10. So we don't even need to calculate x1 and x2.

David Kipping - 2 years, 2 months ago

Log in to reply

Nice, this solves for a right range of values on the right.

Doris Wong - 1 year, 7 months ago

How on earth do you people even think of these ideas on the spot?

Ryan Huynh - 1 year, 10 months ago

Log in to reply

Through practice

Chew-Seong Cheong - 1 year, 10 months ago

darn i tried to use vietas and failed

ALLAN YUAN - 1 year, 6 months ago

Log in to reply

Oops! Hope that the solutions here help.

Chew-Seong Cheong - 1 year, 6 months ago

When x=-10 is substituted the left hand side of the equation is 504 therefore x cannot equal -10

Walker T - 11 months ago

Log in to reply

You are right. The roots are 1 1 and 11 -11 . And the answer needed is the sum of roots (1-11 = - 10) and not x x .

Chew-Seong Cheong - 11 months ago
Nihar Mahajan
Mar 28, 2015

( x + 3 ) ( x + 4 ) ( x + 6 ) ( x + 7 ) = 1120 (x+3)(x+4)(x+6)(x+7)=1120

( x + 3 ) ( x + 7 ) ( x + 4 ) ( x + 6 ) = 1120 (x+3)(x+7)(x+4)(x+6)=1120

( x 2 + 10 x + 21 ) ( x 2 + 10 x + 24 ) = 1120 (x^2+10x+21)(x^2+10x+24)=1120

Let x 2 + 10 x = a x^2+10x=a

( a + 21 ) ( a + 24 ) = 1120 (a+21)(a+24)=1120

a 2 + 45 a + 504 = 1120 a^2+45a+504=1120

a 2 + 45 a 616 = 0 a^2+45a-616=0

( a + 56 ) ( a 11 ) = 0 (a+56)(a-11)=0

a = 56 o r a = 11 \Rightarrow a = -56 \quad or \quad a = 11

When a = 56 x 2 + 10 x + 56 = 0 x = 10 ± 124 2 a=-56 \Rightarrow x^2+10x+56=0 \Rightarrow x=\dfrac{-10\pm\sqrt{-124}}{2} which are complex roots , hence not considered.

When a = 11 x 2 + 10 x 11 = 0 a=11 \Rightarrow x^2+10x-11=0 , the discriminant is positive.

Hence by Vieta's formula , sum of real roots = 10 \huge=\boxed{-10}

I think that the RELATION to the picture in the question is that if we put y = x + 5 y=x+\huge{5} , the whole equation can be reduced to the quadratic equation, which can be further solved very easily.


Also, 5 \huge 5 is the average of 3 , 4 , 6 , 7 3,4,6,7 : listed in the problem.

Sandeep Bhardwaj - 6 years, 2 months ago

Log in to reply

Awesome relation & observation!

Harsh Shrivastava - 6 years, 2 months ago

You can also consider a = x 2 + 10 x + 21 a=x^2+10x+21 .

a ( a + 3 ) = 1120 a 2 + 3 a 1120 = 0 ( a + 35 ) ( a 32 ) = 0 ( x 2 + 10 x + 56 ) ( x 2 + 10 x 11 ) = 0 a(a+3)=1120\\ \implies a^2+3a-1120=0\\ \implies (a+35)(a-32)=0\\ \implies (x^2+10x+56)(x^2+10x-11)=0

The rest is the same as yours.

Prasun Biswas - 6 years, 2 months ago

Log in to reply

Yeah. This is the same method as used in the continuation of the problem!

Mehul Arora - 6 years, 2 months ago

Can't we put x = 1? ( 1 + 3 ) ( 1 + 4 ) ( 1 + 6 ) ( 1 + 7 ) = 4 5 7 8 = 1120 (1+3)(1+4)(1+6)(1+7) = 4 * 5 * 7 * 8 = 1120 Why isn't it correct?

Konstantinos Mihalenas - 5 years, 6 months ago

Log in to reply

the question asks for the sum of all real x. -11 satisfies.

Aareyan Manzoor - 5 years, 6 months ago

Log in to reply

So... Why is x = - 11 better than x = 1

John Jones - 2 years, 9 months ago

not the easiest.. enter x-5 instead of x. then ( x 2 ) ( x 1 ) ( x + 1 ) ( x + 2 ) = 1120 (x-2)(x-1)(x+1)(x+2)=1120 it forms a bi-quadratic whose real root will always have a sum of 0. judging by the quadratic discriminant, it has 2. so n = 1 2 ( x n 5 ) = 0 n = 1 2 ( x n ) = 10 \sum_{n=1}^2 (x_n-5)=0\rightarrow \sum_{n=1}^2 (x_n)=10

Aareyan Manzoor - 6 years, 2 months ago

Can someone explain to me what Vieta's formula is? I am not a mathematical genius so please explain at a commoner level.

John Taylor - 6 years, 2 months ago

Log in to reply

The simplest case is for a quadratic equation, let's say a x 2 + b x + c = 0 ax^{2}+bx+c=0 , with a a , b b and c c as the coefficients respectively, the sum of the two roots is equal to b a -\frac{b}{a} and the product of the two roots is c a \frac{c}{a} . For general cases, please check http://en.wikipedia.org/wiki/Vieta's_formulas

Jessica Wang - 6 years, 2 months ago

Log in to reply

Thanks. I knew the formula but I didn't know that it was called Vieta's formula. I got up to expanding the LHS but I was lost after that.

John Taylor - 6 years, 2 months ago

Log in to reply

@John Taylor i did the same mistake that of yours...

Swarnendu Bhattacharjee - 5 years, 7 months ago

Thanks a lot.....

Shreyas Pai - 5 years, 6 months ago

@John Taylor , This might help you out. ¨ \ddot\smile

Mehul Arora - 6 years, 2 months ago

why can't it be x=1?? then it would be (1+3)(1+4)(1+6)(1+7)=1120

Juan Pablo Diaz Maurtua - 5 years, 7 months ago

Log in to reply

Note that the problem asks for the sum of all real solutions x x . x = 1 x=1 is one of the (real) solutions. There's another one, namely x = ( 11 ) x=(-11) . Their sum is ( 10 ) (-10) , hence the answer.

Prasun Biswas - 5 years, 7 months ago

I agree with you

Mico Arrafi Arrafi - 5 years, 6 months ago

Nice. Same method. Cheers!

Mehul Arora - 6 years, 2 months ago

Log in to reply

¨ ¨ ¨ \ddot\smile\ddot\smile\ddot\smile

Nihar Mahajan - 6 years, 2 months ago

What's the relation of 10 -10 with the picture?

Is it 5 × 2 = 10 5 \times -2 = -10 ? @Mehul Arora

Harsh Shrivastava - 6 years, 2 months ago

Log in to reply

TBH, I do not know that what is the relation. A moderator had probably changed the title of the problem. Initially, the problem was named, "That can't be tough, Can it?"

Mehul Arora - 6 years, 2 months ago

Log in to reply

OK, BTW , Sir Sandeep Bhardawaj found the relation!

Harsh Shrivastava - 6 years, 2 months ago

Log in to reply

@Harsh Shrivastava Yeah! He has a great observation.

Mehul Arora - 6 years, 2 months ago

I think the problem modifier wants to question the people 'where is 5?' Hence to complete the series 3,4,5,6,7 ,he added image of 5. :/

Nihar Mahajan - 6 years, 2 months ago

Log in to reply

That logic is also awesome like Sir Sandeep's logic!

Harsh Shrivastava - 6 years, 2 months ago

I'd also like to point out that 1 also works for the answer, though it says it's wrong.

(1+3)(1+4)(1+6)(1+7)

= 4 x 5 x 7 x 8

= 20 x 56

= (20 x 50) + (20 x 6)

= 1000 + 120

= 1120

I'll point out that I got this answer by guessing, but I'm disappointed that it didn't work.

Flint Watters - 5 years, 5 months ago

Log in to reply

1 is 1 solution. try -11. it asks for the sum.

Aareyan Manzoor - 5 years, 5 months ago

It does work but you stopped just short. The only thing is that for the other solution (-11) the answers would have been -8, -7, -5 and -4 and the product would have been the same.

Zahid Hussain - 2 years ago

-11 not -10 ...correct, also +1 correct

Ghazi Al Ateah - 5 years, 2 months ago

why x is not equal to 1?

niloy debnath - 5 years, 1 month ago

but if we multiply the terms and apply theory of polynomial equations we get x 4 + 20 x 3 + 145 x 2 616 = 0 x^4 +20x^3+145x^2 -616=0 and by theory of polynomial equations sum of roots is given by negative division coefficient of highest power of x upon coefficient of second highest power of x i.e -20

Jus Jaisinghani - 4 years, 11 months ago

Log in to reply

Hi Jus,note that the problem asks for the sum of real roots of the given equation.

Mehul Arora - 4 years, 11 months ago

Log in to reply

OK so basically the method i used gives sum of all real unreal roots thus i got it wrong ? Well if thats the case thn thanks Mehul

Jus Jaisinghani - 4 years, 11 months ago

Log in to reply

@Jus Jaisinghani Yes! Vieta's formula gives the sum of all the real as well as complex roots. Thus, you got it wrong. Glad to be of assistance :)

Mehul Arora - 4 years, 11 months ago

Why we cant put 1? (1+3)(1+4)(1+6)(1+7)=1120 also!!!!!!!!

A Former Brilliant Member - 4 years, 11 months ago

Log in to reply

Hi Shadman, the problem asks for the SUM of the solutions that satisfy the equation above. Indeed, one satisfies the equation, but there are other solutions too.

Mehul Arora - 4 years, 11 months ago

Why can't we do it by subsituting x=y-5

A Former Brilliant Member - 4 years, 7 months ago

cant we just multiply all the monomials and apply the sum of roots formula to it, since the constant has no effect on the sum of roots of any polynomial

Surya Pratap Singh - 4 years, 4 months ago

Log in to reply

No, because that would be the sum of ALL the roots and the questions asks for only the REAL roots

Jim Byrd - 3 years, 6 months ago

I could get x only equal to 1 and -10

Raj Mantri - 4 years, 3 months ago

I also did the same :)

Bibhor Singh - 3 years, 2 months ago

@Christopher Boo Did you modify it? Please tell the logic behind 5!

Nihar Mahajan - 6 years, 2 months ago

Hi, I found a mistake: I check for x=-10 (-10+3)(-10+4)(-10+6)(-10+7)=(-7)(-6)(-4)(-3)= 504 504 1120 504 \neq 1120

I think that there is a mistake: x 2 + 10 x 11 = 0 x^2 + 10x - 11 = 0 d e l t a = b 2 4 a c delta= b^2 - 4ac d e l t a = 100 + 44 = 144 delta= 100 + 44 = 144 d e l t a = 12 \sqrt{delta}= 12 x 1 = 10 + 12 2 = 1 x_1 = \frac {-10 + 12}{2} = 1 x 2 = 10 12 2 = 11 x_2 = \frac {-10 - 12}{2} = -11

And I solved this task: I consider: a= x+5 ( a 2 ) ( a 1 ) ( a + 1 ) ( a + 2 ) = 1120 (a-2)(a-1)(a+1)(a+2)=1120 ( a 2 4 ) ( a 2 1 ) = 1120 (a^2-4)(a^2-1)=1120 a 4 5 a 2 1116 = 0 a^4 - 5a^2 -1116 = 0 I consider: a 2 = t a^2=t , where 0 < t < + 0<t< +\infty t 2 5 t 1116 = 0 t^2 - 5t - 1116= 0 d e l t a = 25 + 1116 × 4 = 4489 delta= 25 + 1116 \times 4 = 4489 d e l t a = 67 \sqrt{delta}= 67 t 1 = 5 + 67 2 = 36 t_1 = \frac {5 + 67}{2} = 36 t 2 = 5 67 2 = 31 t_2 = \frac {5 - 67}{2} = -31 (It's not OK.) So t=36 \ --> a 2 = 36 a^2=36 --> a=6 or a=-6 --> x= -11 or x= 1

What do you think about my solution?

Darek Kotlarz - 5 years, 2 months ago

Log in to reply

"Find the sum of all real satisfying the equation above"

Novril Razenda - 4 years, 11 months ago

Nobody said x = -10. It was -11. So the answer to the original question i.e. sum of the real roots is -10

Zahid Hussain - 2 years ago
Jason Hughes
Mar 29, 2015

We have ( x + 3 ) ( x + 4 ) ( x + 6 ) ( x + 7 ) = 1120. (x+3)(x+4)(x+6)(x+7)=1120. Let x + 5 = z x+5=z . The expression becomes ( z 2 ) ( z 1 ) ( z + 1 ) ( z + 2 ) = 1120. (z-2)(z-1)(z+1)(z+2)=1120.

( z 2 4 ) ( z 2 1 ) = 1120 (z^2-4)(z^2-1)=1120

z 4 5 z 2 + 4 = 1120 z^4-5z^2+4=1120

z 4 5 z 2 1116 = 0 z^4-5z^2-1116=0

( z 2 36 ) ( z 2 + 31 ) = 0 (z^2-36)(z^2+31)=0

The only real values of z z for which this equation is true is z = 6 z=6 and z = 6 z=-6 . x = z 5 x=z-5 so the real x x satisfying this equation are x = 1 x=1 and x = 11 x=-11 .

1 + 11 = 10 1+-11=\boxed{-10}

Kevin Hong
Mar 29, 2015

We factor 1120 into 7x5x2^5=1120.

With a little playing around one can easily find that one positive solution would be 4,5,7 and 8. Where x=1.

We are not done yet. It also appears there could be a negative solution also for this. We then further rule out 2 negatives and 2 positives because that cannot be consistent with the equation. Thus the only solution must have the characteristics of 4 negatives.

Then again there are two possibilities. We already know that the numbers are 4,5,7 and 8 but we don't know which 'way' the equation starts at. In other words does -4=x+3 or -8=x+3? We test the later and find that it works thus giving us x=-11.

We add 1+(-11) together and get -10, our answer.

I solved it in the same way, but how can we know for sure that these two are the only solutions? The problem states that x is a real number, not a whole number

Magne Myhren - 6 years, 2 months ago

Log in to reply

I realized that there was probably only two real integer solutions because of the constraints given. When you see that the factors are in increasing order, they thus must be in intervals of 1 or 2. From there I rationalized that if a number was irrational then it would have a conjugate which would be x less which would invalidate the original condition. The numbers must strictly be increasing in order by steps of 1 or 2 AND be all positive or negative.

Kevin Hong - 6 years, 2 months ago

If x wasn't whole the multiplication wouldn't have a whole answer.

Peter van der Linden - 4 years, 10 months ago

How did your prove that 2 negatives and 2 positives cannot be consistent with the equation ?? Can you explain more explicit please

Pauchen Kho - 4 years, 11 months ago

This is about how I solved it.

Brian Bohan - 2 years, 7 months ago
Lew Sterling Jr
Mar 29, 2015

I love this solution ....easy and straightforward

Oluwatobi Adeyefa - 3 years, 7 months ago

Log in to reply

Thank you so much that you love my solution.

Lew Sterling Jr - 3 years, 7 months ago

1120 = 4 5 6 7 s o i f x + 3 = 4 w e g e t t h e s o l u t i o n x = 1.. N o w t h e c o n s t a n t t e r m i s b y V i e t a s F o r m u l a = 3 4 6 7 1120 = 616 = p r o d u c t = 7 8 11. I f x = ± 7 , ( 7 + 6 ) = 13 o r ( 7 + 4 ) = 3 n o t f a c t o r s o f 1120. I f x = ± 8 , ( x + 7 ) = 15 n o t a f a c t o r 1120 b u t 8 i s . I f x = ± 11 , ( 11 + 4 ) = 15 i s n o t a f a c t o r o f 1120 b u t 11 i s . B u t w i t h x = 8 , t h e p r o d u c t o f f o u r b r a c k e t s i s n o t 1120. B u t w i t h x = 11 , t h e p r o d u c t o f f o u r b r a c k e t s i s 1120. Now the sum of all roots = -(3+4+6+7)= - 20. So the sum of the remaining two roots is = -20 +1 - 11= -10. So other roots can be 1, and -11. That means the expression is a square., but the last is - tive and not a square. So other two roots are not real. Sum of real roots =1 + (-11) = - 10. 1120=4*5*6*7 ~~so ~if~ x+3=4~we~get~ the~ solution~x=1.. \\~Now~the~constant~term~is~by~Vieta's Formula~\\=3*4*6*7-1120=-616=- product=-7*8*11.\\~~\\If~x=\pm~7,~~(7+6)=13 ~or~(-7+4)=-3~not ~ factors~ of~~1120.\\If~x=\pm~8, ~(x+7)=15 ~not ~a~ factor~ ~1120~but~-8~is.\\If~x=\pm~11,~(11+4)=15~is~not~a~factor~of~1120~but -11~is.\\~~\\But~ with ~x=-8, ~the~ product ~of ~four~brackets~is ~not~1120.\\But~ with ~x=-11, ~the~ product ~of ~four~brackets~is~1120.\\\text{Now the sum of all roots = -(3+4+6+7)= - 20. So the sum of the }\\\text{remaining two roots is = -20 +1 - 11= -10. So other roots can be 1, and -11. }\\\text{That means the expression is a square., but the last is - tive and not a square.}\\\text{ So other two roots are not real. Sum of real roots =1 + (-11) = - 10.}

I think you wanted to say 1120 = 4 × 5 × 7 × 8 4 × 5 × 6 × 7 1120 = 4 × 5 × 7 × 8 ≠ 4 × 5 × 6 × 7 . Also, 11 -11 doesn't divide 1120 1120 , but 11 + 4 = 7 -11 + 4 = - 7 does.

Javier Álvarez - 3 years, 4 months ago

The relation with the picture is that there is no x + 5 x+5 term, and hence, one substitutes x = t 5 x = t-5

We have, after the substitution,

( t 2 ) ( t 1 ) ( t + 1 ) ( t + 2 ) = 1120 ( t 2 1 ) ( t 2 4 ) = 1120 \displaystyle (t-2)(t-1)(t+1)(t+2) = 1120 \implies (t^2 -1)(t^2 -4) = 1120

Further substitute s = t 2 s=t^2 . Then,

( s 1 ) ( s 4 ) = 1120 s 2 5 s 1116 = 0 \displaystyle (s-1)(s-4) = 1120 \implies s^2 - 5s -1116 = 0

Since we have real numbers in play here, use the Quadratic Formula, to get:

s = 5 ± 4489 2 = 5 ± 67 2 s = 36 or 31 \displaystyle s = \frac{5 \pm \sqrt{4489}}{2} = \frac{5 \pm 67}{2} \implies s = 36 \text{ or} -31

t 2 = 36 or 31 (rejected, since t 2 > 0 ) t = ± 6 \displaystyle \implies t^2 = 36 \text{ or} -31 \text{ (rejected, since } t^2>0\text{)} \implies t = \pm 6

x = ± 6 5 = 1 or 11 \displaystyle \implies x = \pm 6 - 5 = 1 \text{ or} -11

Hence, x x = 10 \displaystyle \sum_{x\in\Re} x = \boxed{-10}

Jester Koh
Dec 12, 2016

I have a non-standard but relatively quick solution to the problem.

Prime factorise 1120 and you get 2 5 × 5 × 7 2^5 × 5 ×7 which can be rewritten as 4×5×7×8.

Note that this is exactly in the format of the LHS expression, clearly x = 1 x=1 is a solution.

But consider 8 × 7 × 5 × 4 -8×-7×-5×-4 as a possibility thus x = 11 x=-11 is a solution too. Adding both gives you the required answer.

Uahbid Dey
Aug 25, 2015

Yes sir. Same method :)

Mehul Arora - 5 years, 9 months ago
Thicky Bushi
Dec 11, 2015

I found this fairly easy to be solved just by inspection.

The prime factors of 1120 are 2 5 2^{5} , 5 and 7, so it's easy to see that one of the solutions is x = 1. For the other solution, each of the four factors must be the negative equivalents of all the four factors (4, 5, 7 and 8), so x = -11.

Note that x cannot take any other value because each of the factors are in the from of (x + a), and so there is only one combination of real numbers that will produce any real product.

Boon Yang
Mar 29, 2015

Let (x+5)=a . (a-2)(a-1)(a+1)(a+2)=1120.

Why do this? How does it help?

Chris Cheek - 5 years, 3 months ago
Zico Quintina
Nov 22, 2017

As many earlier solutions have noted, we can use the prime factorization of 1120 to quickly find two solutions.

1120 = 2 5 5 7 1120 = 2^5 * 5 * 7

1120 = 32 35 1120 = 32 * 35

1120 = 4 5 7 8 = 8 7 5 4 1120 = 4 * 5 * 7 * 8 = -8 * -7 * -5 * -4

By inspection, we see that the first factorization gives us x = 1, while the second gives us x = -11.

To see that there are no other real solutions, consider the graph of

f ( x ) = ( x + 3 ) ( x + 4 ) ( x + 6 ) ( x + 7 ) f(x) = (x+3) (x+4) (x+6) (x+7)

This is a quartic with a positive leading coefficient and four zeros at -7, -6, -4 and -3 (i.e. it looks like a W.) Thus it has to intersect the line y=1120 at least twice, once to the left of the four zeros and once to the right. Any other intersections of the two graphs would have to occur between the two middle zeros (as between -7 and -6, as well as between -4 and -3, the graph lies below the x-axis.)

By symmetry, the maximum value for f(x) between the middle zeros must occur at x = -5; but f(-5) = 4, so the graphs do not intersect anywhere other than the two points mentioned earlier, and thus x = -11 and x = 1 are indeed the only real solutions to the given equation.

Saúl Huerta
Dec 9, 2019

First, notice that x ≱ 6 x\not\geq6 because:

( 6 + 3 ) ( 6 + 4 ) ( 6 + 6 ) ( 6 + 7 ) = 6 4 + O ( 6 ) + 3 × 4 × 6 × 7 = 1296 + O ( 6 ) + 504 > 1120 (6+3)(6+4)(6+6)(6+7)=6^4+O(6)+3\times4\times6\times7=1296+O(6)+504>1120

Then, after plugging in some values we find that x = 1 x=1 works, and since there is an even number of factors, we can obtain the same result with negative numbers due to the fact that 7 + 3 = 10 7+3=10 , and 6 + 4 = 10 6+4=10 , which can be "inverted" to get the same numbers but with opposite sign when subtracting 10 10 . And to get the same factors we subtract 1 1 again, hence x = 10 1 = 11 x=-10-1=-11 works too.

So the sum is: 11 + 1 = 10 \boxed{-11+1=-10}

Nick Brice
Aug 14, 2017

If you think of the 1120 as the area of a rectangle.

You can arrange the two sides of the rectangle as (x+3)(x+7) and (x+4)(x+6).

When you multiply out the brackets, you are left with x^2+10x+21 as one side and x^2+10x+24 as the other.

The latter can then be written as x^2+10x+(21+3).

Therefore the area of the rectangle can be found to be (x^2+10x+21)^2 + 3(x^2+10x+21) = 1120.

Then by Substituting x^2+10x+21 for y. You get y^2+3y-1120=0.

When solving this equation you get 2 solutions 32 and -35.

Then when you substitute x^2+10x+21 back in and try to solve when = to 32 and -35. The solution for when = 32 is -11 and 1 and there is no real solution for when = -35.

So the answer is -11 + 1 which is -10.

Vu Vincent
Jul 19, 2017

Notice that 1120 = 2 5 × 5 × 7 1120 = 2^5 \times 5 \times 7

If we compare and rearrange with the L H S LHS expression, we can see:

( x + 3 ) ( x + 4 ) ( x + 6 ) ( x + 7 ) = 2 3 × 2 2 × 5 × 7 (x+3)(x+4)(x+6)(x+7)= 2^3 \times 2^2 \times 5 \times 7

( x + 3 ) × ( x + 6 ) × ( x + 4 ) × ( x + 7 ) = 4 × 7 × 5 × 8 \Leftrightarrow (x+3) \times (x+6) \times (x+4) \times (x+7) = 4 \times 7 \times 5 \times 8

This implies that we need to find x x such that it "fits" (or so-called 'symmetric to-') the arrangement on the R H S RHS . We can see that x = 1 \boxed{x = 1} is an answer. However, we must check another way that we can arrange this.

We try to attain x x to a negative value. By doing this we have:

[ 1 ( x + 3 ) ] [ 1 ( x + 4 ) ] [ 1 ( x + 6 ) ] [ 1 ( x + 7 ) ] = 2 3 × 2 2 × 5 × 7 [-1(x+3)][-1(x+4)][-1(x+6)][-1(x+7)]= 2^3 \times 2^2 \times 5 \times 7

( x 3 ) ( x 4 ) ( x 6 ) ( x 7 ) = 4 × 7 × 5 × 8 (-x-3)(-x-4)(-x-6)(-x-7)= 4 \times 7 \times 5 \times 8

Eyeballing for x x gives us x = 11 \boxed{x = -11}

The reason why we do this is because of these two possible factorizations of 1120 1120 :

  • 2 3 × 2 2 × 5 × 7 -2^3 \times -2^2 \times 5 \times 7

  • 2 3 × 2 2 × 5 × 7 -2^3 \times -2^2 \times -5 \times -7

which have negative values that could accord to the products of the terms in the L H S LHS polynomial, but gives the value of a positive integer. We see that the first option on the list is not symmetric to the L H S LHS polynomial, hence not a feasible factorization. Second option is possible because it is symmetric to the L H S LHS polynomial since its just every term in the product times 1 -1 .

Therefore, the sum of all solution to x x is 11 + 1 = 10 \large \boxed{ -11 + 1 = -10}

Viktor Jarenfors
Nov 30, 2016

Simply separate 1120 into primes. Note: 5, 7, 2^2, 2^3. Since there are even number of factors two solutions exists; the above values for positive and negative. Which only occurs for 1 and -11; 1 + - 11 = -10.

Komal Vasudeva
Aug 27, 2016

By analysis, we can see that it will have 2 real roots . Let they be a and b. As product has 4 terms , negative sign gets compensated. We have , (a+3)=-(b+7) , (a+4)=-(b+6) , and so on. Solving any one, we get a+b=-10.

Zach Bian
Aug 22, 2016

Probably should have solved it using algebra... but that's no fun. Instead, here's a numerical method.

 1
 2
 3
 4
 5
 6
 7
 8
 9
10
11
12
13
14
15
16
17
18
19
20
21
22
23
24
25
26
27
28
29
30
31
32
from math import *

# Newton's method for finding roots
# Original function
f = lambda x: (x+3)*(x+4)*(x+6)*(x+7)-1120
# Derivative:
fp = lambda x: (x+4)*(x+6)*(x+7)+(x+3)*((x+6)*(x+7)+(x+4)*(2*x+13))

roots={}
guesses=[]

i=-617         # Highest coefficient = 616 -> loose upper bound
while i <= 617:
    guesses.append(i)
    i+=0.5

for guess in guesses:
    val = f(guess)
    deriv = fp(guess)
    if deriv == 0:
        continue
    nextguess = -val/deriv
    err = 1e-5

    while fabs(nextguess-guess)>err and fabs(nextguess-guess)<1e5:
        guess = nextguess
        nextguess = guess-f(guess)/fp(guess)

    roots[round(nextguess, 5)] = 0

for root in roots.keys():
    print(root)

Louis Noizet
Aug 12, 2016

( x + 3 ) ( x + 4 ) ( x + 6 ) ( x + 7 ) = ( x + 3 ) ( x + 7 ) ( x + 4 ) ( x + 6 ) (x+3)(x+4)(x+6)(x+7)=(x+3)(x+7)(x+4)(x+6)

( x + 3 ) ( x + 4 ) ( x + 6 ) ( x + 7 ) = ( x + 5 2 ) ( x + 5 + 2 ) ( x + 5 1 ) ( x + 5 + 1 ) (x+3)(x+4)(x+6)(x+7)=(x+5-2)(x+5+2)(x+5-1)(x+5+1)

( x + 3 ) ( x + 4 ) ( x + 6 ) ( x + 7 ) = ( ( x + 5 ) 2 4 ) ( ( x + 5 ) 2 1 ) (x+3)(x+4)(x+6)(x+7)=\left((x+5)^2-4)((x+5)^2-1\right)

1120 = 32 35 = ( 32 ) ( 35 ) 1120 = 32\cdot 35=(-32)\cdot(-35)

So we've obviously got 4 solutions, the two solutions of ( x + 5 ) 2 = 36 (x+5)^2 = 36 , which are 1 and -11, and the two solution of ( x + 5 ) 2 = 31 (x+5)^2=-31 which aren't real.

Since this is an equation of degree four, they are all the solutions there are.

Novril Razenda
Jul 6, 2016

Make the equation above becomes : ( 4 ) ( 5 ) ( 7 ) ( 8 ) = 1120 4)(5)(7)(8)= 1120 or ( 8 ) ( 7 ) ( 5 ) ( 4 ) = 1120 -8)(-7)(-5)(-4)=1120

Therefore ( x = 1 ) x=1) and ( x = 11 ) x=-11) So , The Answer is ( 1 11 = 10 ) 1-11= -10)

Consider that if (x+3)=-(y+7), x and y real roots, we have the same numbers being multiply with negative signal, but as (-1)^4=1 it gives the same number.

x+3=-y-7

x+y=-7-3

x+y=-10

take (x+5=y)....then we shall get the equation simlified as, (y^4)-(y^2)-1116=0..........now by middle term factorisation we get (y^2)=36...i.e y=6,-6....x=y-5=1,-11 therefore the sum is -10

Caio Pompéia
Mar 31, 2015

If y=x+5

Then the equation became

(y-2)(y-1)(y+1)(y+2)=1120

(y-2)(y+2) (y-1)(y+1)=(y-2^2)(y^2-1^2)

y^4-5y^2+4=1120

Bhaskara for y^2

y^2=36 or y^2=-31

Only real solutions

y=6 or y=-6

So x=1 or x=-11

Finally

-11+1=-10

The function y=(x+3)(x+4)(x+6)(x+7) has -5 as its medium point and the function is symetric around it. The function has 4 roots. For values bigger than --3 or less than -7 increases. For most of values of y there are two solutions that are at the same distance from -5 and the sum of them is always -10. Someone could proof it mathematically.

Carolus Magnus - 5 years, 3 months ago

2 pending reports

Vote up reports you agree with

×

Problem Loading...

Note Loading...

Set Loading...